2 votos

Una pregunta sobre la diferenciación

$g(x)=\sin(1/x) , g(0)=0$

Definir $G(x)=\int _{0}^{x}g\left( t\right) dt$

Mostrar $G'(0)=g(0)$

Utilizo la definición

$$G'(0) = \lim _{h\rightarrow 0}\dfrac {G\left( h\right) -G\left( 0\right) } {h}$$

$$=\lim _{h\rightarrow 0}\dfrac {\int _{0}^{h}g\left( t\right) dt} {h}$$

$$=\lim _{h\rightarrow 0}\dfrac {\int _{0}^{h}\sin(1/t) dt} {h}$$

No tengo ni idea del siguiente paso

¿Puedes darme alguna pista?

¡¡¡Gracias!!!

0 votos

Esto es complicado. Utilice el hecho de que $$\frac{d} {dx} x^{2}\cos\left(\frac{1}{x}\right)=2x\cos\left(\frac{1}{x}\right)+\sin\left(\frac{1}{x}\right)$$

0 votos

Ver una pregunta similar pero más difícil math.stackexchange.com/questions/2063078/ Eche también un vistazo a los enfoques alternativos que se dan en esta pregunta math.stackexchange.com/q/1551332/72031

0 votos

¡¡¡su sugerencia es muy útil,gracias!!!

3voto

Paramanand Singh Puntos 13338

Ampliando mi comentario, dejemos $F(x) =x^{2}\cos(1/x),F(0)=0,f(x)=2x\cos(1/x),f(0)=0$ . Podemos observar que $F$ es diferenciable en todas partes, $f$ es continua en todas partes y $g$ es continua en todas partes excepto en $x=0$ y $g$ está acotado. Es fácil ver que $F'(x) =f(x)+g(x) $ para todos $x$ y $f(x) +g(x) $ es integrable de Riemann en cualquier intervalo cerrado y acotado. Así, por el segundo teorema fundamental del cálculo tenemos $$F(x) - F(0)=\int_{0}^{x}\{f(t)+g(t)\}\,dt$$ o $$x^{2}\cos\left(\frac{1}{x}\right)=\int_{0}^{x}f(t)\,dt+\int_{0}^{x}g(t)\,dt$$ o $$x\cos\left(\frac{1}{x}\right)= \frac{1}{x}\int_{0}^{x}f(t)\,dt+\frac{1}{x}\int_{0}^{x}g(t)\,dt$$ Tomando los límites como $x\to 0$ obtenemos $$0=f(0)+\lim_{x\to 0}\frac{1}{x}\int_{0}^{x}g(t)\,dt$$ donde el límite del primer término de la derecha se evalúa como $f(0)$ debido al primer teorema fundamental del cálculo y al hecho de que $f$ es continua en $x=0$ . Desde $f(0)=0$ se deduce que $$G'(0)=\lim_{x\to 0}\frac{1}{x}\int_{0}^{x}g(t)\,dt=0=g(0)$$

0voto

Qwerty Puntos 86

¡Utiliza L'Hopital!

Recordando que $${d\over dx}\int _{f(x)}^{g(x)}h(t)dt=g'(x)h(g(x))-f'(x)h(f(x))$$

0 votos

¿Cómo es que el límite (y por tanto $G'(0)$ ) no está indefinido en 0?

0 votos

@alphacapture ¿Esto iba dirigido a mí? Vale... Porque $G'(0)$ no está definido en $0$ el PO lo ha definido explícitamente como $0$ .

0 votos

Esto no se aplica en este caso. La pregunta no es tan trivial como parece. Su respuesta es errónea. - 1

i-Ciencias.com

I-Ciencias es una comunidad de estudiantes y amantes de la ciencia en la que puedes resolver tus problemas y dudas.
Puedes consultar las preguntas de otros usuarios, hacer tus propias preguntas o resolver las de los demás.

Powered by:

X